Wolves

This topic has expert replies
User avatar
Legendary Member
Posts: 1255
Joined: Fri Nov 07, 2008 2:08 pm
Location: St. Louis
Thanked: 312 times
Followed by:90 members

by Tani » Fri Sep 09, 2011 2:43 pm
C is out of scope. We have no information about human encroachment on the wolves' territory.
Tani Wolff

Junior | Next Rank: 30 Posts
Posts: 13
Joined: Fri Sep 24, 2010 12:01 pm

by pearl_rafter » Fri Sep 09, 2011 5:37 pm
As the stem says
'what may be inferred'
I think then only possible choice is D.

User avatar
Newbie | Next Rank: 10 Posts
Posts: 8
Joined: Mon Sep 05, 2011 10:52 pm

by akashamrev » Tue Sep 13, 2011 5:14 am
I would go with option:D,but am still confused with option D and option C can anyone please help me..??

User avatar
Legendary Member
Posts: 1255
Joined: Fri Nov 07, 2008 2:08 pm
Location: St. Louis
Thanked: 312 times
Followed by:90 members

by Tani » Tue Sep 13, 2011 12:11 pm
We have no information about human encroachment on wolf territory. Therefore, although it may be true, it is not a valid inference. Given our lack of information, it is equally possible that humans have backed off and created more territory for the wolves.
Tani Wolff

Master | Next Rank: 500 Posts
Posts: 416
Joined: Thu Jul 28, 2011 12:48 am
Thanked: 28 times
Followed by:6 members

by gunjan1208 » Tue Sep 13, 2011 9:18 pm
I chose D. This is a great question. Kudos.

Senior | Next Rank: 100 Posts
Posts: 85
Joined: Tue Sep 02, 2008 12:13 am
Thanked: 1 times
GMAT Score:650

by vzzai » Tue Sep 13, 2011 9:35 pm
Thank you, Good one. Took a while but finally could arrive at the answer!
Would you rate this as a 700+ question?
Thank you,
Vj

Master | Next Rank: 500 Posts
Posts: 197
Joined: Thu Sep 15, 2011 10:22 am
Thanked: 6 times
Followed by:2 members

by parul9 » Tue Sep 20, 2011 8:07 am
I debated between D and E for some time.
Then, I read D the nth time, and thought of it as a conclusion of the passage, it seemed to suggest that the author was saying that wolf hunting in the in the fringes can continue (as if approving of wolf hunting), while the passage tone never seemed to give such a hint.
And hence I ended up answering as E... :(

I am still not convinced with D... though it seems the best possible choice here, but it is not a balanced conclusion.
Can someone please help with my dilemma here?

User avatar
Legendary Member
Posts: 1255
Joined: Fri Nov 07, 2008 2:08 pm
Location: St. Louis
Thanked: 312 times
Followed by:90 members

by Tani » Tue Sep 20, 2011 8:25 am
E is a 180. The stimulus tells us that reproduction rates vary according to population density. That means density must vary,
Tani Wolff

Master | Next Rank: 500 Posts
Posts: 197
Joined: Thu Sep 15, 2011 10:22 am
Thanked: 6 times
Followed by:2 members

by parul9 » Tue Sep 20, 2011 8:41 am
Thanks Tani.

But when a question asks for "conclusion", then shouldn't the same be derivable from the passage without extrapolation unlike option D?
Though I get that this is the closest option, but isn't there a level of ambiguity there in this choice? Or do I need to change my thought process towards these qsns???

User avatar
Legendary Member
Posts: 1255
Joined: Fri Nov 07, 2008 2:08 pm
Location: St. Louis
Thanked: 312 times
Followed by:90 members

by Tani » Tue Sep 20, 2011 9:18 am
This question asks what conclusion could be properly drawn. It is not looking for a conclusion that is already in the stimulus. That makes it an inference question and the task is, in fact, to extrapolate what MUST BE TRUE based on the stimulus.
Tani Wolff

Master | Next Rank: 500 Posts
Posts: 197
Joined: Thu Sep 15, 2011 10:22 am
Thanked: 6 times
Followed by:2 members

by parul9 » Tue Sep 20, 2011 9:42 am
Noted... thanks!

Master | Next Rank: 500 Posts
Posts: 152
Joined: Wed Mar 12, 2008 4:36 pm
Thanked: 8 times
Followed by:2 members

by artistocrat » Wed Sep 21, 2011 4:27 pm
DanaJ wrote:Source: Beat The GMAT Practice Questions

Wolves can double their numbers every year. They do not do so, however, if the area in which they live is already full and the territory of each pack borders on the territory of other packs on all sides. Lone wolves, unable to establish a territory near their place of origin, disperse to a less desirable habitat and often are hunted by people.

Which of the following conclusions may be properly drawn from the information in the passage above?

A. The number of wolves in any area suitable for wolves may be predicted to quadruple in two years.
B. Wolves are ejected from the pack to wander singly as lone wolves when the number of wolves in the pack has doubled.
C. The amount of territory suitable for wolves has been diminishing in recent years as a result of human encroachment, with a resulting diminution in the wolf population.
D. Wolf hunting can continue at a moderate rate on the fringes of a good habitat for wolves without a decrease in the average yearly wolf population in that territory.
E. The wolf population density in a given area of wolf territory does not vary with the yearly rate of wolf reproduction.
Process of Elimination if answer is not evident:

A. What if the saturation point is reached after one 2 year cycle?
B. A given wolf population may double again before it ejects lone wolves.
C. Out of Scope: Who knows the impact of human encroachment? The argument doesn't even show the impact of hunting.
D. The answer for reasons mentioned by others already.
E. The argument does not say that density and rate are not linked.

Instructors and students, please correct me if I am wrong.

Newbie | Next Rank: 10 Posts
Posts: 2
Joined: Sun Sep 04, 2011 6:41 pm

by juanbinaghi » Fri Oct 07, 2011 2:11 pm
..
Last edited by juanbinaghi on Sat Jun 29, 2013 11:01 am, edited 1 time in total.

User avatar
Senior | Next Rank: 100 Posts
Posts: 46
Joined: Fri Aug 19, 2011 12:13 am
Location: Delhi, India
Followed by:1 members

by romitvsingh » Thu Oct 20, 2011 9:18 am
never seen such a question in OG, are these real GMAT problems

Senior | Next Rank: 100 Posts
Posts: 90
Joined: Thu Nov 05, 2009 9:14 am
Thanked: 5 times
Followed by:3 members

by immaculatesahai » Fri Nov 04, 2011 11:45 pm
+1 for D

A is wrong. It goes against the latter sentences in the stimulus. A person maybe misguided into selecting A if he is in a hurry and pays attention only to the first line.

B is wrong. The passage does not supply information regarding whether wolves are ejected from the pack or they leave at will.

C is wrong. No information regarding human encroachment in stimulus.

D is correct.

E is too strong. All the passage says is that "Wolves can double their numbers every year". The stimulus does not supply any information regarding what the original population of wolves is. If there is scope for the population to grow and double within the area, then this WILL infact increase the population density.

Hope this helps.